Difference between revisions of "2013 AIME II Problems/Problem 14"
(Problem 14) |
|||
Line 1: | Line 1: | ||
For positive integers <math>n</math> and <math>k</math>, let <math>f(n, k)</math> be the remainder when <math>n</math> is divided by <math>k</math>, and for <math>n > 1</math> let <math>F(n) = \max_{\substack{1\le k\le \frac{n}{2}}} f(n, k)</math>. Find the remainder when <math>\sum\limits_{n=20}^{100} F(n)</math> is divided by <math>1000</math>. | For positive integers <math>n</math> and <math>k</math>, let <math>f(n, k)</math> be the remainder when <math>n</math> is divided by <math>k</math>, and for <math>n > 1</math> let <math>F(n) = \max_{\substack{1\le k\le \frac{n}{2}}} f(n, k)</math>. Find the remainder when <math>\sum\limits_{n=20}^{100} F(n)</math> is divided by <math>1000</math>. | ||
+ | |||
+ | ==Solution== | ||
+ | |||
+ | ====Solution without strict proof==== | ||
+ | |||
+ | We can find that | ||
+ | |||
+ | <math>20\equiv 6 \pmod{7}</math> | ||
+ | |||
+ | <math>21\equiv 5 \pmod{8}</math> | ||
+ | |||
+ | <math>22\equiv 6 \pmod{8}</math> | ||
+ | |||
+ | <math>23\equiv 7 \pmod{8}</math> | ||
+ | |||
+ | <math>24\equiv 6 \pmod{9}</math> | ||
+ | |||
+ | <math>25\equiv 7 \pmod{9}</math> | ||
+ | |||
+ | <math>26\equiv 8 \pmod{9}</math> | ||
+ | |||
+ | Observing these and we can find that the reminders are in groups of three continuous integers, considering this is true, and we get | ||
+ | |||
+ | <math>99\equiv 31 \pmod{34}</math> | ||
+ | |||
+ | <math>100\equiv 32 \pmod{34}</math> | ||
+ | |||
+ | So the sum is <math>5+3\times(6+...+31)+32\times 2=1512</math>, so the answer is <math>\boxed{512}</math> |
Revision as of 03:48, 5 April 2013
For positive integers and , let be the remainder when is divided by , and for let . Find the remainder when is divided by .
Solution
Solution without strict proof
We can find that
Observing these and we can find that the reminders are in groups of three continuous integers, considering this is true, and we get
So the sum is , so the answer is